Force de Lorentz entre 2 charges en mouvement
ACTION # REACTION
par Jean-Louis Naudin
Click here for the english version
Créé le 5 Décembre 2002 - JLN Labs - Dernière mise à jour le 28 mars 2004
Toutes les informations et schémas sont publiés gratuitement ( freeware ) et sont destinés à un usage personnel et non commercial

All informations and diagrams are published freely (freeware) and are intended for a private use and a non commercial use.


Cette expérience est décrite dans "Le cours de Physique de Richard Feynman" - Electromagnetisme 2, Chap: 26-2 par Addison-Wesley Publishing company et collection InterEdition - ISBN 2-7296-0029-9.

A - Champ magnétique créé par une particule chargée en mouvement

Une particule chargée en mouvement produit un champ magnétique qui tourne autour de la direction de son déplacement.

B - Force de Coulomb et Force de Lorentz entre 2 particules chargées en mouvement

Prenons le cas de deux particules chargées ( positivement ) de charges respectives Q1 et Q2, se déplaçant avec une vitesse V1 et V2 avec sur des trajectoires convergentes et perpendiculaires. La position initiale de Q1 est déphasée de telle sorte que Q1 ne rentre pas en collision avec Q2 mais passe derrière celle-ci.
Regardez attentivement l'animation ci-dessus. On constate qu'il n'y a pas de champ magnétique le long de la trajectoire de la particule Q
1. Observons ce qui se produit lorsque la particule Q1 arrive à proximité de la particule Q2 ( dernière étape de l'animation ) :

  • Il n'y a pas de champ magnétique dans l'axe de Q1, alors Q2 n'est soumise qu'à la force électrostatique de Coulomb produite par le champ électrique de Q1,

  • Par contre, Q1 est sous l'influence du champ magnétique produit par Q2. Il y a dans ce cas 2 forces en présence : la force électrostatique produite par le champ électrique de Q2 et la force magnétique de Lorentz produite par son propre déplacement dans le champ magnétique de Q2.

Les forces électriques de Coulomb sont égales et opposées, elles vérifient le 3ieme principe de Newton, par contre il n'y a qu'une force magnétique de Lorentz sur Q1 et aucune force magnétique sur Q2.

Nous avons ici, une Force d'ACTION SANS Force de REACTION... Dans ce cas, le 3ieme principe de Newton n'est pas vérifié ici.

Le centre de masse des particules Q1 et Q2 accélère dans une direction préférentielle sans qu'il n'y ait aucune force externe sur le système...

Note : Pour être en accord avec le 3ieme principe de Newton, il serait nécessaire de prendre en compte les moments des champs magnétique et électrique.

Documents de référence :

  • "Le cours de Physique de Richard Feynman" - Electromagnetisme 2 - Collection InterEditions

  • La Physique en MP - PC 1re Année - Volume 3 - Electricité ( électromagnétisme ) par Pierre Alais et Michel Hulin - Librairie Armand Colin.

Cliquez ici pour les résultats des tests sur le propulseur à condensateur asymétrique


Email : JNaudin509@aol.com


Retour à la page sommaire